2015 AMC 12A Problems/Problem 14

Revision as of 22:32, 4 February 2015 by Suli (talk | contribs) (Created page with "==Problem 14== What is the value of <math>a</math> for which <math>\frac{1}{\text{log}_2a} + \frac{1}{\text{log}_3a} + \frac{1}{\text{log}_4a} = 1</math>? <math> \textbf{(A)}\ ...")
(diff) ← Older revision | Latest revision (diff) | Newer revision → (diff)

Problem 14

What is the value of $a$ for which $\frac{1}{\text{log}_2a} + \frac{1}{\text{log}_3a} + \frac{1}{\text{log}_4a} = 1$?

$\textbf{(A)}\ 9\qquad\textbf{(B)}\ 12\qquad\textbf{(C)}\ 18\qquad\textbf{(D)}}\ 24\qquad\textbf{(E)}\ 36$ (Error compiling LaTeX. Unknown error_msg)

Solution

We use the change of base formula to show that \[\log_a b = \dfrac{\log_b b}{\log_b a} = \dfrac{1}{\log_b a}.\] Thus, our equation becomes \[\log_a 2 + \log_a 3 + \log_a 4 = 1,\] which becomes after combining: \[\log_a 24 = 1.\] Hence $a = 24$, and the answer is $\textbf{(D)}.$